Exercices

Exercice 1   Pour chacun des ensembles de réels suivants :

$\displaystyle \left\{(-1)^n ,\;n\in\mathbb{N}\right\} ,\;
\left\{(-1)^n/n ,\;n\in\mathbb{N}^*\right\} ,\;
\left\{(-1)^nn ,\;n\in\mathbb{N}\right\} ,\;
$

$\displaystyle \left\{\frac{n+1}{n+2} ,\;n\in\mathbb{N}\right\} ,\;
\left\{\fr...
...b{N}\right\} ,\;
\left\{\frac{2n+(-1)^n}{n+2} ,\;n\in\mathbb{N}\right\} ,\;
$

$\displaystyle \left\{\frac{m+n}{m+2n} , m,n\in\mathbb{N}^*\right\} ,\;
\left...
...bb{N}^*\right\} ,\;
\left\{\frac{m-n}{m+2n} , m,n\in\mathbb{N}^*\right\}\;.
$

  1. L'ensemble est-il majoré ? minoré ?
  2. L'ensemble admet-il un plus grand élément ? un plus petit élément ?
  3. Déterminer la borne supérieure et la borne inférieure de l'ensemble.

Exercice 2   On considère les ensembles de réels suivants :

$\displaystyle \left\{x\in\mathbb{R} ,\;\vert x\vert> 1\right\} ,\;
\left\{x\i...
...bb{R} ,\;x^2\leqslant 1\right\} ,\;
\left\{x\in\mathbb{R} ,\;x^3<1 \right\}
$

$\displaystyle \left\{x\in\mathbb{R}^* ,\;1/x\leqslant 1\right\} ,\;
\left\{x\...
...thbb{R}^* ,\;1/x> 1\right\} ,\;
\left\{x\in\mathbb{R}^* ,\;1/x^2< 1\right\}
$

$\displaystyle \left\{x\in\mathbb{R}^{+}, \sin x \leqslant 0\right\} ,\;
\left...
...{x\in\mathbb{R}^{+*}, \vert\sin\frac{1}{x}\vert<\frac{\sqrt{2}}{2}\right\}\;.
$

  1. Ecrire l'ensemble comme un intervalle ou une réunion d'intervalles disjoints.
  2. L'ensemble est-il majoré ? minoré ?
  3. L'ensemble admet-il un plus grand élément ? un plus petit élément ?
  4. Déterminer la borne supérieure et la borne inférieure de l'ensemble.

Exercice 3   Soient $ A$ et $ B$ deux parties non vides et bornées de $ \mathbb{R}$.
  1. Montrer que $ A\subset B$ implique $ \sup(A)\leqslant\sup(B)$ et $ \inf(A)\geqslant \inf(B)$.
  2. Montrer que $ A\cup B$ admet une borne supérieure et une borne inférieure finies. Montrer que

    $\displaystyle \sup(A\cup B) = \max\{\sup(A),\sup(B)\}$   et$\displaystyle \quad
\inf(A\cup B) = \min\{\inf(A),\inf(B)\}\;.
$

  3. Montrer que si l'intersection $ A\cap B$ est non vide, alors elle admet une borne supérieure et une borne inférieure finies. Montrer que

    $\displaystyle \sup(A\cap B) \leqslant \min\{\sup(A),\sup(B)\}$   et$\displaystyle \quad
\inf(A\cap B) \geqslant \max\{\inf(A),\inf(B)\}\;.
$

  4. On note $ A+B=\{a+b ,\;a\in A, b\in B\}$. Montrer que $ A+B$ admet une borne supérieure et une borne inférieure finies. Montrer que

    $\displaystyle \sup(A+B) = \sup(A)+\sup(B)$   et$\displaystyle \quad
\inf(A+B) = \inf(A)+\inf(B)\;.
$

  5. On note $ AB=\{ab ,\;a\in A ,\; b\in B\}$. Montrer que $ AB$ admet une borne supérieure et une borne inférieure. Montrer que, si $ A$ et $ B$ sont inclus dans $ \mathbb{R}^+$, alors

    $\displaystyle \sup(AB) = \sup(A) \sup(B)$   et$\displaystyle \quad
\inf(AB) = \inf(A) \inf(B)\;.
$

Exercice 4   Soient $ A$ et $ B$ deux intervalles de $ \mathbb{R}$.
  1. Montrer que $ A\cap B$ est un intervalle.
  2. Montrer que si $ A\cap B$ est non vide, alors $ A\cup B$ est un intervalle.
  3. Montrer par un exemple que $ A\cup B$ peut être un intervalle même si $ A\cap B$ est vide.
  4. On note $ A+B=\{a+b ,\;a\in A, b\in B\}$. Montrer que $ A+B$ est un intervalle.
  5. On note $ AB=\{ab ,\;a\in A ,\; b\in B\}$. Montrer que $ AB$ est un intervalle.

Exercice 5   Soient $ x$ et $ y$ deux rationnels distincts tels que $ \sqrt{x}$ et $ \sqrt{y}$ soient irrationnels.
  1. On considère les deux réels $ \sqrt{x}+\sqrt{y}$ et $ \sqrt{x}-\sqrt{y}$. Montrer que leur produit est rationnel, leur somme irrationnelle. En déduire qu'ils sont irrationnels.
  2. Soient $ r$ et $ s$ deux rationnels. Montrer que $ r\sqrt{x}+s \sqrt{y}$ est irrationnel.
  3. Montrer par des exemples que $ \sqrt{x}\sqrt{y}$ peut être rationnel ou irrationnel.
  4. Montrer que les réels suivants sont irrationnels.

    $\displaystyle 1+\sqrt{2} ,\;
\sqrt{2}+\sqrt{3} ,\;
(\sqrt{2}+\sqrt{3})^2 ,\;
$

    $\displaystyle \sqrt{2}+\sqrt{3}-\sqrt{6} ,\;
\sqrt{2}-\sqrt{3}+\sqrt{6} ,\;
(\sqrt{2}+\sqrt{3}+\sqrt{6})^2\;.
$

Exercice 6   Soit $ I$ un intervalle de $ \mathbb{R}$ contenant deux points distincts. Montrer que $ I$ contient :
  1. une infinité de rationnels,
  2. une infinité d'irrationnels,
  3. une infinité de nombres décimaux,
  4. une infinité de nombres multiples entiers d'une certaine puissance négative de $ 2$ (nombres dyadiques).


         © UJF Grenoble, 2011                              Mentions légales